Prove that for all $n$ $gcd(n, x_n)=1$, given $x_{n+1}=2(x_n)^2-1$ and $x_1=2$












5












$begingroup$


I have a sequence $x_{n+1} = 2(x_n)^2-1$; first values are $2, 7, 97, 18817,dots$
I noticed that if prime $p$ divides $x_n$, then $x_{n+1} equiv -1pmod p$ and for all $k>n+1$, $x_kequiv 1pmod p$. But I have no idea what to do next.










share|cite|improve this question











$endgroup$












  • $begingroup$
    $x_n$ is A002812 at the OEIS. It seems that the least prime factor of $x_n$ is larger than (or equal to) $2^{n+1}-1$. See the comments at A002812 where the question whether for $n>1$, equality holds iff $2^{n+1}-1$ is a Mersenne prime is raised.
    $endgroup$
    – René Gy
    Dec 7 '18 at 20:36










  • $begingroup$
    $ x_{n+1} = sum_k {2^n choose 2k} 3^k2^{2^n-2k}$ . Not sure that this explicit expression can be useful here.
    $endgroup$
    – René Gy
    Dec 7 '18 at 21:32












  • $begingroup$
    Also $x_n-1=3cdot2^{2n-1}cdot prod_{j=2}^{n-2}x_j^2$ for $n>2$. But it is not clear how this could be used.
    $endgroup$
    – René Gy
    Dec 8 '18 at 12:01








  • 1




    $begingroup$
    Would it be possible to show that for $n>1$, any prime divisor $p$ of $x_n$ verify $p equiv pm 1 bmod {2^n}$ ? That would imply a proof of the OP.
    $endgroup$
    – René Gy
    Dec 8 '18 at 12:42
















5












$begingroup$


I have a sequence $x_{n+1} = 2(x_n)^2-1$; first values are $2, 7, 97, 18817,dots$
I noticed that if prime $p$ divides $x_n$, then $x_{n+1} equiv -1pmod p$ and for all $k>n+1$, $x_kequiv 1pmod p$. But I have no idea what to do next.










share|cite|improve this question











$endgroup$












  • $begingroup$
    $x_n$ is A002812 at the OEIS. It seems that the least prime factor of $x_n$ is larger than (or equal to) $2^{n+1}-1$. See the comments at A002812 where the question whether for $n>1$, equality holds iff $2^{n+1}-1$ is a Mersenne prime is raised.
    $endgroup$
    – René Gy
    Dec 7 '18 at 20:36










  • $begingroup$
    $ x_{n+1} = sum_k {2^n choose 2k} 3^k2^{2^n-2k}$ . Not sure that this explicit expression can be useful here.
    $endgroup$
    – René Gy
    Dec 7 '18 at 21:32












  • $begingroup$
    Also $x_n-1=3cdot2^{2n-1}cdot prod_{j=2}^{n-2}x_j^2$ for $n>2$. But it is not clear how this could be used.
    $endgroup$
    – René Gy
    Dec 8 '18 at 12:01








  • 1




    $begingroup$
    Would it be possible to show that for $n>1$, any prime divisor $p$ of $x_n$ verify $p equiv pm 1 bmod {2^n}$ ? That would imply a proof of the OP.
    $endgroup$
    – René Gy
    Dec 8 '18 at 12:42














5












5








5


2



$begingroup$


I have a sequence $x_{n+1} = 2(x_n)^2-1$; first values are $2, 7, 97, 18817,dots$
I noticed that if prime $p$ divides $x_n$, then $x_{n+1} equiv -1pmod p$ and for all $k>n+1$, $x_kequiv 1pmod p$. But I have no idea what to do next.










share|cite|improve this question











$endgroup$




I have a sequence $x_{n+1} = 2(x_n)^2-1$; first values are $2, 7, 97, 18817,dots$
I noticed that if prime $p$ divides $x_n$, then $x_{n+1} equiv -1pmod p$ and for all $k>n+1$, $x_kequiv 1pmod p$. But I have no idea what to do next.







elementary-number-theory coprime






share|cite|improve this question















share|cite|improve this question













share|cite|improve this question




share|cite|improve this question








edited Dec 7 '18 at 17:22









user10354138

7,4322925




7,4322925










asked Dec 7 '18 at 17:01









IEVA ŠAPOVALOVAITĖIEVA ŠAPOVALOVAITĖ

634




634












  • $begingroup$
    $x_n$ is A002812 at the OEIS. It seems that the least prime factor of $x_n$ is larger than (or equal to) $2^{n+1}-1$. See the comments at A002812 where the question whether for $n>1$, equality holds iff $2^{n+1}-1$ is a Mersenne prime is raised.
    $endgroup$
    – René Gy
    Dec 7 '18 at 20:36










  • $begingroup$
    $ x_{n+1} = sum_k {2^n choose 2k} 3^k2^{2^n-2k}$ . Not sure that this explicit expression can be useful here.
    $endgroup$
    – René Gy
    Dec 7 '18 at 21:32












  • $begingroup$
    Also $x_n-1=3cdot2^{2n-1}cdot prod_{j=2}^{n-2}x_j^2$ for $n>2$. But it is not clear how this could be used.
    $endgroup$
    – René Gy
    Dec 8 '18 at 12:01








  • 1




    $begingroup$
    Would it be possible to show that for $n>1$, any prime divisor $p$ of $x_n$ verify $p equiv pm 1 bmod {2^n}$ ? That would imply a proof of the OP.
    $endgroup$
    – René Gy
    Dec 8 '18 at 12:42


















  • $begingroup$
    $x_n$ is A002812 at the OEIS. It seems that the least prime factor of $x_n$ is larger than (or equal to) $2^{n+1}-1$. See the comments at A002812 where the question whether for $n>1$, equality holds iff $2^{n+1}-1$ is a Mersenne prime is raised.
    $endgroup$
    – René Gy
    Dec 7 '18 at 20:36










  • $begingroup$
    $ x_{n+1} = sum_k {2^n choose 2k} 3^k2^{2^n-2k}$ . Not sure that this explicit expression can be useful here.
    $endgroup$
    – René Gy
    Dec 7 '18 at 21:32












  • $begingroup$
    Also $x_n-1=3cdot2^{2n-1}cdot prod_{j=2}^{n-2}x_j^2$ for $n>2$. But it is not clear how this could be used.
    $endgroup$
    – René Gy
    Dec 8 '18 at 12:01








  • 1




    $begingroup$
    Would it be possible to show that for $n>1$, any prime divisor $p$ of $x_n$ verify $p equiv pm 1 bmod {2^n}$ ? That would imply a proof of the OP.
    $endgroup$
    – René Gy
    Dec 8 '18 at 12:42
















$begingroup$
$x_n$ is A002812 at the OEIS. It seems that the least prime factor of $x_n$ is larger than (or equal to) $2^{n+1}-1$. See the comments at A002812 where the question whether for $n>1$, equality holds iff $2^{n+1}-1$ is a Mersenne prime is raised.
$endgroup$
– René Gy
Dec 7 '18 at 20:36




$begingroup$
$x_n$ is A002812 at the OEIS. It seems that the least prime factor of $x_n$ is larger than (or equal to) $2^{n+1}-1$. See the comments at A002812 where the question whether for $n>1$, equality holds iff $2^{n+1}-1$ is a Mersenne prime is raised.
$endgroup$
– René Gy
Dec 7 '18 at 20:36












$begingroup$
$ x_{n+1} = sum_k {2^n choose 2k} 3^k2^{2^n-2k}$ . Not sure that this explicit expression can be useful here.
$endgroup$
– René Gy
Dec 7 '18 at 21:32






$begingroup$
$ x_{n+1} = sum_k {2^n choose 2k} 3^k2^{2^n-2k}$ . Not sure that this explicit expression can be useful here.
$endgroup$
– René Gy
Dec 7 '18 at 21:32














$begingroup$
Also $x_n-1=3cdot2^{2n-1}cdot prod_{j=2}^{n-2}x_j^2$ for $n>2$. But it is not clear how this could be used.
$endgroup$
– René Gy
Dec 8 '18 at 12:01






$begingroup$
Also $x_n-1=3cdot2^{2n-1}cdot prod_{j=2}^{n-2}x_j^2$ for $n>2$. But it is not clear how this could be used.
$endgroup$
– René Gy
Dec 8 '18 at 12:01






1




1




$begingroup$
Would it be possible to show that for $n>1$, any prime divisor $p$ of $x_n$ verify $p equiv pm 1 bmod {2^n}$ ? That would imply a proof of the OP.
$endgroup$
– René Gy
Dec 8 '18 at 12:42




$begingroup$
Would it be possible to show that for $n>1$, any prime divisor $p$ of $x_n$ verify $p equiv pm 1 bmod {2^n}$ ? That would imply a proof of the OP.
$endgroup$
– René Gy
Dec 8 '18 at 12:42










3 Answers
3






active

oldest

votes


















2












$begingroup$

We can write



$$x_n=frac{1}{2}left[tau^{2^{n-1}}+tau^{-2^{n-1}}right]$$



where $tau=2+sqrt{3}$. Now, given an odd prime $p$, $p|x_n$ if and only if



$$tau^{2^{n-1}}+tau^{-2^{n-1}}=0$$



(where we work in $mathbb{F}_{p^2}$). This reduces to



$$tau^{2^n}= -1.$$



Let $k$ be the multiplicative order of $tau$ (the smallest positive integer so $tau^k=1$). Then we have



$$tau^{2^{n+1}}=1,tau^k=1implies tau^{gcdleft(2^{n+1},kright)}=1,$$



which implies $k|2^{n+1}$ as $k$ is minimal. However, if $k=2^m$ with $mleq n$ then



$$-1=tau^{2^n}=left(tau^{2^m}right)^{2^{n-m}}=1^{2^{n-m}}=1,$$



a contradiction, so $k=2^{n+1}$. A result of this is that $2^{n+1}$ divides the order of $mathbb{F}_{p^2}^{times}$, which is $p^2-1$; this implies that $2^n$ divides $ppm 1$; in particular,



$$pgeq 2^n-1>n$$



(where the second condition holds if $n>1$). Only $p=2$ divides $x_n=1$, and if $p=2$, then $p|x_n$ iff $n=1$. So, any prime $p$ that divides $x_n$ is greater than $n$, which finishes our proof.






share|cite|improve this answer









$endgroup$













  • $begingroup$
    In order to be able to work in $mathbb{F}_{p^2}$, we need $tau in mathbb{F}_{p^2}$, right? For this, don't we need that $3$ be quadratic non-residue $bmod p$? If yes, how is this justified?
    $endgroup$
    – René Gy
    Dec 16 '18 at 11:25










  • $begingroup$
    @RenéGy If $3$ is a quadratic residue $bmod p$, then $2+sqrt{3}in mathbb{F}_psubsetmathbb{F}_{p^2}$.
    $endgroup$
    – Carl Schildkraut
    Dec 16 '18 at 17:45



















1












$begingroup$

I'll do my usual playing around
and see what happens.



tl;dr - Nothing but dead ends.



If
$x_{n+1}
= 2(x_n)^2-1
$

then
$x_{n+1} -1
= 2(x_n)^2-2
= 2(x_n^2-1)
= 2(x_n-1)(x_n+1)
$
.



Therefore,
if $y_n =x_n-1$ then
$y_{n+1}
=2y_n(y_n+2)
$
.



Since
$x_1 = 2$,
$y_1 = 1$.



Therefore
$y_2 = 2cdot 1cdot 3 = 6$,
$y_3 = 2cdot 6cdot 8= 96$,
$y_4 = 2cdot 96cdot 98 = 18616$,
and
$y_5 = 2cdot 18616cdot 18618 = 693185376$.



If we can show that
$n | y_n$,
then
$(n, x_n) = 1$
since,
if $d|n$ and $d|x_n$
and $d ge 2$
then
$d | n | y_n$
so
$d | (x_n-1)$
implies
$d | 1$.



Unfortunately,
this doesn't work
as $y_5$ shows.



Next try:
see if can find
$a_n, b_n$ such that
$na_n-x_nb_n = 1$.
This will show that
$(n, x_n) = 1$.



If
$na_n-x_nb_n = 1$
and
$(n+1)a_{n+1}-x_{n+1}b_{n+1} = 1$
then



$begin{array}\
1
&=(n+1)a_{n+1}-(2x_n^2-1)b_{n+1}\
&=na_{n+1}+a_{n+1}-2x_n^2b_{n+1}+b_{n+1}\
&=na_{n}+n(a_{n+1}-a_n)+a_{n+1}-x_n(2x_nb_{n+1})+b_{n+1}\
&=x_nb_n+1+n(a_{n+1}-a_n)+a_{n+1}-x_n(2x_nb_{n+1})+b_{n+1}\
&=1+n(a_{n+1}-a_n)+a_{n+1}-x_n(2x_nb_{n+1}+b_{n})+b_{n+1}\
text{so}\
0
&=n(a_{n+1}-a_n)+a_{n+1}-x_n(2x_nb_{n+1}+b_{n})+b_{n+1}\
end{array}
$



and I don't know where to go from here.



Hope someone else can make use of these.






share|cite|improve this answer









$endgroup$





















    1












    $begingroup$

    This is the pigeonhole principle. Among odd primes $p,$ there are solutions to $2t^2 - 1 equiv 0 pmod p$ if and only if $p equiv pm 1 pmod 8.$



    For such a prime $p,$ there are some $frac{p+1}{2}$ distinct values $pmod p$ of $2x^2 - 1.$ Therefore, taking $p geq 17,$ if we take, say, $p-5$ steps of your sequence, there are just two possibilities: either



    (I) there is a repetition of some value $pmod p$ that is not one of $-1,0,1.$ In this case, the sequence repeats again and again, forever. Thus, this prime $p$ is never a factor of any of your $x_n.$ OR



    (II) at least one of $-1,0,1$ occurs by the $p-5$ deadline. Note that $0$ must come first out of these three, as $2 cdot 1^2 - 1 equiv 2 cdot (-1)^2 - 1 equiv 1 pmod p,$ while the only way to get $-1$ is $2 cdot 0^2 - 1 equiv -1 pmod p.$



    That's it. If $p$ will ever be a factor of any $x_n,$ it is because, say, $x_{p-2} equiv 1 pmod p$



    jagy@phobeusjunior:~$ ./mse 
    7
    2 0 -1

    17
    2 7 12 15 7

    23
    2 7 5 3 17 2

    31
    2 7 4 0 -1

    41
    2 7 15 39 7

    47
    2 7 3 17 13 8 33 15 26 35
    5 2

    71
    2 7 26 2

    73
    2 7 24 56 66 24

    79
    2 7 18 15 54 64 54

    89
    2 7 8 38 39 15 4 31 52 67
    77 20 87 7

    97
    2 7 0 -1


    good primes
    2
    7
    31
    97


    =======================



    The good primes up to 30,000 are



    2
    7
    31
    97
    127
    607
    8191
    12289
    22783


    ===========






    share|cite|improve this answer









    $endgroup$













      Your Answer





      StackExchange.ifUsing("editor", function () {
      return StackExchange.using("mathjaxEditing", function () {
      StackExchange.MarkdownEditor.creationCallbacks.add(function (editor, postfix) {
      StackExchange.mathjaxEditing.prepareWmdForMathJax(editor, postfix, [["$", "$"], ["\\(","\\)"]]);
      });
      });
      }, "mathjax-editing");

      StackExchange.ready(function() {
      var channelOptions = {
      tags: "".split(" "),
      id: "69"
      };
      initTagRenderer("".split(" "), "".split(" "), channelOptions);

      StackExchange.using("externalEditor", function() {
      // Have to fire editor after snippets, if snippets enabled
      if (StackExchange.settings.snippets.snippetsEnabled) {
      StackExchange.using("snippets", function() {
      createEditor();
      });
      }
      else {
      createEditor();
      }
      });

      function createEditor() {
      StackExchange.prepareEditor({
      heartbeatType: 'answer',
      autoActivateHeartbeat: false,
      convertImagesToLinks: true,
      noModals: true,
      showLowRepImageUploadWarning: true,
      reputationToPostImages: 10,
      bindNavPrevention: true,
      postfix: "",
      imageUploader: {
      brandingHtml: "Powered by u003ca class="icon-imgur-white" href="https://imgur.com/"u003eu003c/au003e",
      contentPolicyHtml: "User contributions licensed under u003ca href="https://creativecommons.org/licenses/by-sa/3.0/"u003ecc by-sa 3.0 with attribution requiredu003c/au003e u003ca href="https://stackoverflow.com/legal/content-policy"u003e(content policy)u003c/au003e",
      allowUrls: true
      },
      noCode: true, onDemand: true,
      discardSelector: ".discard-answer"
      ,immediatelyShowMarkdownHelp:true
      });


      }
      });














      draft saved

      draft discarded


















      StackExchange.ready(
      function () {
      StackExchange.openid.initPostLogin('.new-post-login', 'https%3a%2f%2fmath.stackexchange.com%2fquestions%2f3030131%2fprove-that-for-all-n-gcdn-x-n-1-given-x-n1-2x-n2-1-and-x-1-2%23new-answer', 'question_page');
      }
      );

      Post as a guest















      Required, but never shown

























      3 Answers
      3






      active

      oldest

      votes








      3 Answers
      3






      active

      oldest

      votes









      active

      oldest

      votes






      active

      oldest

      votes









      2












      $begingroup$

      We can write



      $$x_n=frac{1}{2}left[tau^{2^{n-1}}+tau^{-2^{n-1}}right]$$



      where $tau=2+sqrt{3}$. Now, given an odd prime $p$, $p|x_n$ if and only if



      $$tau^{2^{n-1}}+tau^{-2^{n-1}}=0$$



      (where we work in $mathbb{F}_{p^2}$). This reduces to



      $$tau^{2^n}= -1.$$



      Let $k$ be the multiplicative order of $tau$ (the smallest positive integer so $tau^k=1$). Then we have



      $$tau^{2^{n+1}}=1,tau^k=1implies tau^{gcdleft(2^{n+1},kright)}=1,$$



      which implies $k|2^{n+1}$ as $k$ is minimal. However, if $k=2^m$ with $mleq n$ then



      $$-1=tau^{2^n}=left(tau^{2^m}right)^{2^{n-m}}=1^{2^{n-m}}=1,$$



      a contradiction, so $k=2^{n+1}$. A result of this is that $2^{n+1}$ divides the order of $mathbb{F}_{p^2}^{times}$, which is $p^2-1$; this implies that $2^n$ divides $ppm 1$; in particular,



      $$pgeq 2^n-1>n$$



      (where the second condition holds if $n>1$). Only $p=2$ divides $x_n=1$, and if $p=2$, then $p|x_n$ iff $n=1$. So, any prime $p$ that divides $x_n$ is greater than $n$, which finishes our proof.






      share|cite|improve this answer









      $endgroup$













      • $begingroup$
        In order to be able to work in $mathbb{F}_{p^2}$, we need $tau in mathbb{F}_{p^2}$, right? For this, don't we need that $3$ be quadratic non-residue $bmod p$? If yes, how is this justified?
        $endgroup$
        – René Gy
        Dec 16 '18 at 11:25










      • $begingroup$
        @RenéGy If $3$ is a quadratic residue $bmod p$, then $2+sqrt{3}in mathbb{F}_psubsetmathbb{F}_{p^2}$.
        $endgroup$
        – Carl Schildkraut
        Dec 16 '18 at 17:45
















      2












      $begingroup$

      We can write



      $$x_n=frac{1}{2}left[tau^{2^{n-1}}+tau^{-2^{n-1}}right]$$



      where $tau=2+sqrt{3}$. Now, given an odd prime $p$, $p|x_n$ if and only if



      $$tau^{2^{n-1}}+tau^{-2^{n-1}}=0$$



      (where we work in $mathbb{F}_{p^2}$). This reduces to



      $$tau^{2^n}= -1.$$



      Let $k$ be the multiplicative order of $tau$ (the smallest positive integer so $tau^k=1$). Then we have



      $$tau^{2^{n+1}}=1,tau^k=1implies tau^{gcdleft(2^{n+1},kright)}=1,$$



      which implies $k|2^{n+1}$ as $k$ is minimal. However, if $k=2^m$ with $mleq n$ then



      $$-1=tau^{2^n}=left(tau^{2^m}right)^{2^{n-m}}=1^{2^{n-m}}=1,$$



      a contradiction, so $k=2^{n+1}$. A result of this is that $2^{n+1}$ divides the order of $mathbb{F}_{p^2}^{times}$, which is $p^2-1$; this implies that $2^n$ divides $ppm 1$; in particular,



      $$pgeq 2^n-1>n$$



      (where the second condition holds if $n>1$). Only $p=2$ divides $x_n=1$, and if $p=2$, then $p|x_n$ iff $n=1$. So, any prime $p$ that divides $x_n$ is greater than $n$, which finishes our proof.






      share|cite|improve this answer









      $endgroup$













      • $begingroup$
        In order to be able to work in $mathbb{F}_{p^2}$, we need $tau in mathbb{F}_{p^2}$, right? For this, don't we need that $3$ be quadratic non-residue $bmod p$? If yes, how is this justified?
        $endgroup$
        – René Gy
        Dec 16 '18 at 11:25










      • $begingroup$
        @RenéGy If $3$ is a quadratic residue $bmod p$, then $2+sqrt{3}in mathbb{F}_psubsetmathbb{F}_{p^2}$.
        $endgroup$
        – Carl Schildkraut
        Dec 16 '18 at 17:45














      2












      2








      2





      $begingroup$

      We can write



      $$x_n=frac{1}{2}left[tau^{2^{n-1}}+tau^{-2^{n-1}}right]$$



      where $tau=2+sqrt{3}$. Now, given an odd prime $p$, $p|x_n$ if and only if



      $$tau^{2^{n-1}}+tau^{-2^{n-1}}=0$$



      (where we work in $mathbb{F}_{p^2}$). This reduces to



      $$tau^{2^n}= -1.$$



      Let $k$ be the multiplicative order of $tau$ (the smallest positive integer so $tau^k=1$). Then we have



      $$tau^{2^{n+1}}=1,tau^k=1implies tau^{gcdleft(2^{n+1},kright)}=1,$$



      which implies $k|2^{n+1}$ as $k$ is minimal. However, if $k=2^m$ with $mleq n$ then



      $$-1=tau^{2^n}=left(tau^{2^m}right)^{2^{n-m}}=1^{2^{n-m}}=1,$$



      a contradiction, so $k=2^{n+1}$. A result of this is that $2^{n+1}$ divides the order of $mathbb{F}_{p^2}^{times}$, which is $p^2-1$; this implies that $2^n$ divides $ppm 1$; in particular,



      $$pgeq 2^n-1>n$$



      (where the second condition holds if $n>1$). Only $p=2$ divides $x_n=1$, and if $p=2$, then $p|x_n$ iff $n=1$. So, any prime $p$ that divides $x_n$ is greater than $n$, which finishes our proof.






      share|cite|improve this answer









      $endgroup$



      We can write



      $$x_n=frac{1}{2}left[tau^{2^{n-1}}+tau^{-2^{n-1}}right]$$



      where $tau=2+sqrt{3}$. Now, given an odd prime $p$, $p|x_n$ if and only if



      $$tau^{2^{n-1}}+tau^{-2^{n-1}}=0$$



      (where we work in $mathbb{F}_{p^2}$). This reduces to



      $$tau^{2^n}= -1.$$



      Let $k$ be the multiplicative order of $tau$ (the smallest positive integer so $tau^k=1$). Then we have



      $$tau^{2^{n+1}}=1,tau^k=1implies tau^{gcdleft(2^{n+1},kright)}=1,$$



      which implies $k|2^{n+1}$ as $k$ is minimal. However, if $k=2^m$ with $mleq n$ then



      $$-1=tau^{2^n}=left(tau^{2^m}right)^{2^{n-m}}=1^{2^{n-m}}=1,$$



      a contradiction, so $k=2^{n+1}$. A result of this is that $2^{n+1}$ divides the order of $mathbb{F}_{p^2}^{times}$, which is $p^2-1$; this implies that $2^n$ divides $ppm 1$; in particular,



      $$pgeq 2^n-1>n$$



      (where the second condition holds if $n>1$). Only $p=2$ divides $x_n=1$, and if $p=2$, then $p|x_n$ iff $n=1$. So, any prime $p$ that divides $x_n$ is greater than $n$, which finishes our proof.







      share|cite|improve this answer












      share|cite|improve this answer



      share|cite|improve this answer










      answered Dec 11 '18 at 2:49









      Carl SchildkrautCarl Schildkraut

      11.7k11443




      11.7k11443












      • $begingroup$
        In order to be able to work in $mathbb{F}_{p^2}$, we need $tau in mathbb{F}_{p^2}$, right? For this, don't we need that $3$ be quadratic non-residue $bmod p$? If yes, how is this justified?
        $endgroup$
        – René Gy
        Dec 16 '18 at 11:25










      • $begingroup$
        @RenéGy If $3$ is a quadratic residue $bmod p$, then $2+sqrt{3}in mathbb{F}_psubsetmathbb{F}_{p^2}$.
        $endgroup$
        – Carl Schildkraut
        Dec 16 '18 at 17:45


















      • $begingroup$
        In order to be able to work in $mathbb{F}_{p^2}$, we need $tau in mathbb{F}_{p^2}$, right? For this, don't we need that $3$ be quadratic non-residue $bmod p$? If yes, how is this justified?
        $endgroup$
        – René Gy
        Dec 16 '18 at 11:25










      • $begingroup$
        @RenéGy If $3$ is a quadratic residue $bmod p$, then $2+sqrt{3}in mathbb{F}_psubsetmathbb{F}_{p^2}$.
        $endgroup$
        – Carl Schildkraut
        Dec 16 '18 at 17:45
















      $begingroup$
      In order to be able to work in $mathbb{F}_{p^2}$, we need $tau in mathbb{F}_{p^2}$, right? For this, don't we need that $3$ be quadratic non-residue $bmod p$? If yes, how is this justified?
      $endgroup$
      – René Gy
      Dec 16 '18 at 11:25




      $begingroup$
      In order to be able to work in $mathbb{F}_{p^2}$, we need $tau in mathbb{F}_{p^2}$, right? For this, don't we need that $3$ be quadratic non-residue $bmod p$? If yes, how is this justified?
      $endgroup$
      – René Gy
      Dec 16 '18 at 11:25












      $begingroup$
      @RenéGy If $3$ is a quadratic residue $bmod p$, then $2+sqrt{3}in mathbb{F}_psubsetmathbb{F}_{p^2}$.
      $endgroup$
      – Carl Schildkraut
      Dec 16 '18 at 17:45




      $begingroup$
      @RenéGy If $3$ is a quadratic residue $bmod p$, then $2+sqrt{3}in mathbb{F}_psubsetmathbb{F}_{p^2}$.
      $endgroup$
      – Carl Schildkraut
      Dec 16 '18 at 17:45











      1












      $begingroup$

      I'll do my usual playing around
      and see what happens.



      tl;dr - Nothing but dead ends.



      If
      $x_{n+1}
      = 2(x_n)^2-1
      $

      then
      $x_{n+1} -1
      = 2(x_n)^2-2
      = 2(x_n^2-1)
      = 2(x_n-1)(x_n+1)
      $
      .



      Therefore,
      if $y_n =x_n-1$ then
      $y_{n+1}
      =2y_n(y_n+2)
      $
      .



      Since
      $x_1 = 2$,
      $y_1 = 1$.



      Therefore
      $y_2 = 2cdot 1cdot 3 = 6$,
      $y_3 = 2cdot 6cdot 8= 96$,
      $y_4 = 2cdot 96cdot 98 = 18616$,
      and
      $y_5 = 2cdot 18616cdot 18618 = 693185376$.



      If we can show that
      $n | y_n$,
      then
      $(n, x_n) = 1$
      since,
      if $d|n$ and $d|x_n$
      and $d ge 2$
      then
      $d | n | y_n$
      so
      $d | (x_n-1)$
      implies
      $d | 1$.



      Unfortunately,
      this doesn't work
      as $y_5$ shows.



      Next try:
      see if can find
      $a_n, b_n$ such that
      $na_n-x_nb_n = 1$.
      This will show that
      $(n, x_n) = 1$.



      If
      $na_n-x_nb_n = 1$
      and
      $(n+1)a_{n+1}-x_{n+1}b_{n+1} = 1$
      then



      $begin{array}\
      1
      &=(n+1)a_{n+1}-(2x_n^2-1)b_{n+1}\
      &=na_{n+1}+a_{n+1}-2x_n^2b_{n+1}+b_{n+1}\
      &=na_{n}+n(a_{n+1}-a_n)+a_{n+1}-x_n(2x_nb_{n+1})+b_{n+1}\
      &=x_nb_n+1+n(a_{n+1}-a_n)+a_{n+1}-x_n(2x_nb_{n+1})+b_{n+1}\
      &=1+n(a_{n+1}-a_n)+a_{n+1}-x_n(2x_nb_{n+1}+b_{n})+b_{n+1}\
      text{so}\
      0
      &=n(a_{n+1}-a_n)+a_{n+1}-x_n(2x_nb_{n+1}+b_{n})+b_{n+1}\
      end{array}
      $



      and I don't know where to go from here.



      Hope someone else can make use of these.






      share|cite|improve this answer









      $endgroup$


















        1












        $begingroup$

        I'll do my usual playing around
        and see what happens.



        tl;dr - Nothing but dead ends.



        If
        $x_{n+1}
        = 2(x_n)^2-1
        $

        then
        $x_{n+1} -1
        = 2(x_n)^2-2
        = 2(x_n^2-1)
        = 2(x_n-1)(x_n+1)
        $
        .



        Therefore,
        if $y_n =x_n-1$ then
        $y_{n+1}
        =2y_n(y_n+2)
        $
        .



        Since
        $x_1 = 2$,
        $y_1 = 1$.



        Therefore
        $y_2 = 2cdot 1cdot 3 = 6$,
        $y_3 = 2cdot 6cdot 8= 96$,
        $y_4 = 2cdot 96cdot 98 = 18616$,
        and
        $y_5 = 2cdot 18616cdot 18618 = 693185376$.



        If we can show that
        $n | y_n$,
        then
        $(n, x_n) = 1$
        since,
        if $d|n$ and $d|x_n$
        and $d ge 2$
        then
        $d | n | y_n$
        so
        $d | (x_n-1)$
        implies
        $d | 1$.



        Unfortunately,
        this doesn't work
        as $y_5$ shows.



        Next try:
        see if can find
        $a_n, b_n$ such that
        $na_n-x_nb_n = 1$.
        This will show that
        $(n, x_n) = 1$.



        If
        $na_n-x_nb_n = 1$
        and
        $(n+1)a_{n+1}-x_{n+1}b_{n+1} = 1$
        then



        $begin{array}\
        1
        &=(n+1)a_{n+1}-(2x_n^2-1)b_{n+1}\
        &=na_{n+1}+a_{n+1}-2x_n^2b_{n+1}+b_{n+1}\
        &=na_{n}+n(a_{n+1}-a_n)+a_{n+1}-x_n(2x_nb_{n+1})+b_{n+1}\
        &=x_nb_n+1+n(a_{n+1}-a_n)+a_{n+1}-x_n(2x_nb_{n+1})+b_{n+1}\
        &=1+n(a_{n+1}-a_n)+a_{n+1}-x_n(2x_nb_{n+1}+b_{n})+b_{n+1}\
        text{so}\
        0
        &=n(a_{n+1}-a_n)+a_{n+1}-x_n(2x_nb_{n+1}+b_{n})+b_{n+1}\
        end{array}
        $



        and I don't know where to go from here.



        Hope someone else can make use of these.






        share|cite|improve this answer









        $endgroup$
















          1












          1








          1





          $begingroup$

          I'll do my usual playing around
          and see what happens.



          tl;dr - Nothing but dead ends.



          If
          $x_{n+1}
          = 2(x_n)^2-1
          $

          then
          $x_{n+1} -1
          = 2(x_n)^2-2
          = 2(x_n^2-1)
          = 2(x_n-1)(x_n+1)
          $
          .



          Therefore,
          if $y_n =x_n-1$ then
          $y_{n+1}
          =2y_n(y_n+2)
          $
          .



          Since
          $x_1 = 2$,
          $y_1 = 1$.



          Therefore
          $y_2 = 2cdot 1cdot 3 = 6$,
          $y_3 = 2cdot 6cdot 8= 96$,
          $y_4 = 2cdot 96cdot 98 = 18616$,
          and
          $y_5 = 2cdot 18616cdot 18618 = 693185376$.



          If we can show that
          $n | y_n$,
          then
          $(n, x_n) = 1$
          since,
          if $d|n$ and $d|x_n$
          and $d ge 2$
          then
          $d | n | y_n$
          so
          $d | (x_n-1)$
          implies
          $d | 1$.



          Unfortunately,
          this doesn't work
          as $y_5$ shows.



          Next try:
          see if can find
          $a_n, b_n$ such that
          $na_n-x_nb_n = 1$.
          This will show that
          $(n, x_n) = 1$.



          If
          $na_n-x_nb_n = 1$
          and
          $(n+1)a_{n+1}-x_{n+1}b_{n+1} = 1$
          then



          $begin{array}\
          1
          &=(n+1)a_{n+1}-(2x_n^2-1)b_{n+1}\
          &=na_{n+1}+a_{n+1}-2x_n^2b_{n+1}+b_{n+1}\
          &=na_{n}+n(a_{n+1}-a_n)+a_{n+1}-x_n(2x_nb_{n+1})+b_{n+1}\
          &=x_nb_n+1+n(a_{n+1}-a_n)+a_{n+1}-x_n(2x_nb_{n+1})+b_{n+1}\
          &=1+n(a_{n+1}-a_n)+a_{n+1}-x_n(2x_nb_{n+1}+b_{n})+b_{n+1}\
          text{so}\
          0
          &=n(a_{n+1}-a_n)+a_{n+1}-x_n(2x_nb_{n+1}+b_{n})+b_{n+1}\
          end{array}
          $



          and I don't know where to go from here.



          Hope someone else can make use of these.






          share|cite|improve this answer









          $endgroup$



          I'll do my usual playing around
          and see what happens.



          tl;dr - Nothing but dead ends.



          If
          $x_{n+1}
          = 2(x_n)^2-1
          $

          then
          $x_{n+1} -1
          = 2(x_n)^2-2
          = 2(x_n^2-1)
          = 2(x_n-1)(x_n+1)
          $
          .



          Therefore,
          if $y_n =x_n-1$ then
          $y_{n+1}
          =2y_n(y_n+2)
          $
          .



          Since
          $x_1 = 2$,
          $y_1 = 1$.



          Therefore
          $y_2 = 2cdot 1cdot 3 = 6$,
          $y_3 = 2cdot 6cdot 8= 96$,
          $y_4 = 2cdot 96cdot 98 = 18616$,
          and
          $y_5 = 2cdot 18616cdot 18618 = 693185376$.



          If we can show that
          $n | y_n$,
          then
          $(n, x_n) = 1$
          since,
          if $d|n$ and $d|x_n$
          and $d ge 2$
          then
          $d | n | y_n$
          so
          $d | (x_n-1)$
          implies
          $d | 1$.



          Unfortunately,
          this doesn't work
          as $y_5$ shows.



          Next try:
          see if can find
          $a_n, b_n$ such that
          $na_n-x_nb_n = 1$.
          This will show that
          $(n, x_n) = 1$.



          If
          $na_n-x_nb_n = 1$
          and
          $(n+1)a_{n+1}-x_{n+1}b_{n+1} = 1$
          then



          $begin{array}\
          1
          &=(n+1)a_{n+1}-(2x_n^2-1)b_{n+1}\
          &=na_{n+1}+a_{n+1}-2x_n^2b_{n+1}+b_{n+1}\
          &=na_{n}+n(a_{n+1}-a_n)+a_{n+1}-x_n(2x_nb_{n+1})+b_{n+1}\
          &=x_nb_n+1+n(a_{n+1}-a_n)+a_{n+1}-x_n(2x_nb_{n+1})+b_{n+1}\
          &=1+n(a_{n+1}-a_n)+a_{n+1}-x_n(2x_nb_{n+1}+b_{n})+b_{n+1}\
          text{so}\
          0
          &=n(a_{n+1}-a_n)+a_{n+1}-x_n(2x_nb_{n+1}+b_{n})+b_{n+1}\
          end{array}
          $



          and I don't know where to go from here.



          Hope someone else can make use of these.







          share|cite|improve this answer












          share|cite|improve this answer



          share|cite|improve this answer










          answered Dec 7 '18 at 18:51









          marty cohenmarty cohen

          74.3k549128




          74.3k549128























              1












              $begingroup$

              This is the pigeonhole principle. Among odd primes $p,$ there are solutions to $2t^2 - 1 equiv 0 pmod p$ if and only if $p equiv pm 1 pmod 8.$



              For such a prime $p,$ there are some $frac{p+1}{2}$ distinct values $pmod p$ of $2x^2 - 1.$ Therefore, taking $p geq 17,$ if we take, say, $p-5$ steps of your sequence, there are just two possibilities: either



              (I) there is a repetition of some value $pmod p$ that is not one of $-1,0,1.$ In this case, the sequence repeats again and again, forever. Thus, this prime $p$ is never a factor of any of your $x_n.$ OR



              (II) at least one of $-1,0,1$ occurs by the $p-5$ deadline. Note that $0$ must come first out of these three, as $2 cdot 1^2 - 1 equiv 2 cdot (-1)^2 - 1 equiv 1 pmod p,$ while the only way to get $-1$ is $2 cdot 0^2 - 1 equiv -1 pmod p.$



              That's it. If $p$ will ever be a factor of any $x_n,$ it is because, say, $x_{p-2} equiv 1 pmod p$



              jagy@phobeusjunior:~$ ./mse 
              7
              2 0 -1

              17
              2 7 12 15 7

              23
              2 7 5 3 17 2

              31
              2 7 4 0 -1

              41
              2 7 15 39 7

              47
              2 7 3 17 13 8 33 15 26 35
              5 2

              71
              2 7 26 2

              73
              2 7 24 56 66 24

              79
              2 7 18 15 54 64 54

              89
              2 7 8 38 39 15 4 31 52 67
              77 20 87 7

              97
              2 7 0 -1


              good primes
              2
              7
              31
              97


              =======================



              The good primes up to 30,000 are



              2
              7
              31
              97
              127
              607
              8191
              12289
              22783


              ===========






              share|cite|improve this answer









              $endgroup$


















                1












                $begingroup$

                This is the pigeonhole principle. Among odd primes $p,$ there are solutions to $2t^2 - 1 equiv 0 pmod p$ if and only if $p equiv pm 1 pmod 8.$



                For such a prime $p,$ there are some $frac{p+1}{2}$ distinct values $pmod p$ of $2x^2 - 1.$ Therefore, taking $p geq 17,$ if we take, say, $p-5$ steps of your sequence, there are just two possibilities: either



                (I) there is a repetition of some value $pmod p$ that is not one of $-1,0,1.$ In this case, the sequence repeats again and again, forever. Thus, this prime $p$ is never a factor of any of your $x_n.$ OR



                (II) at least one of $-1,0,1$ occurs by the $p-5$ deadline. Note that $0$ must come first out of these three, as $2 cdot 1^2 - 1 equiv 2 cdot (-1)^2 - 1 equiv 1 pmod p,$ while the only way to get $-1$ is $2 cdot 0^2 - 1 equiv -1 pmod p.$



                That's it. If $p$ will ever be a factor of any $x_n,$ it is because, say, $x_{p-2} equiv 1 pmod p$



                jagy@phobeusjunior:~$ ./mse 
                7
                2 0 -1

                17
                2 7 12 15 7

                23
                2 7 5 3 17 2

                31
                2 7 4 0 -1

                41
                2 7 15 39 7

                47
                2 7 3 17 13 8 33 15 26 35
                5 2

                71
                2 7 26 2

                73
                2 7 24 56 66 24

                79
                2 7 18 15 54 64 54

                89
                2 7 8 38 39 15 4 31 52 67
                77 20 87 7

                97
                2 7 0 -1


                good primes
                2
                7
                31
                97


                =======================



                The good primes up to 30,000 are



                2
                7
                31
                97
                127
                607
                8191
                12289
                22783


                ===========






                share|cite|improve this answer









                $endgroup$
















                  1












                  1








                  1





                  $begingroup$

                  This is the pigeonhole principle. Among odd primes $p,$ there are solutions to $2t^2 - 1 equiv 0 pmod p$ if and only if $p equiv pm 1 pmod 8.$



                  For such a prime $p,$ there are some $frac{p+1}{2}$ distinct values $pmod p$ of $2x^2 - 1.$ Therefore, taking $p geq 17,$ if we take, say, $p-5$ steps of your sequence, there are just two possibilities: either



                  (I) there is a repetition of some value $pmod p$ that is not one of $-1,0,1.$ In this case, the sequence repeats again and again, forever. Thus, this prime $p$ is never a factor of any of your $x_n.$ OR



                  (II) at least one of $-1,0,1$ occurs by the $p-5$ deadline. Note that $0$ must come first out of these three, as $2 cdot 1^2 - 1 equiv 2 cdot (-1)^2 - 1 equiv 1 pmod p,$ while the only way to get $-1$ is $2 cdot 0^2 - 1 equiv -1 pmod p.$



                  That's it. If $p$ will ever be a factor of any $x_n,$ it is because, say, $x_{p-2} equiv 1 pmod p$



                  jagy@phobeusjunior:~$ ./mse 
                  7
                  2 0 -1

                  17
                  2 7 12 15 7

                  23
                  2 7 5 3 17 2

                  31
                  2 7 4 0 -1

                  41
                  2 7 15 39 7

                  47
                  2 7 3 17 13 8 33 15 26 35
                  5 2

                  71
                  2 7 26 2

                  73
                  2 7 24 56 66 24

                  79
                  2 7 18 15 54 64 54

                  89
                  2 7 8 38 39 15 4 31 52 67
                  77 20 87 7

                  97
                  2 7 0 -1


                  good primes
                  2
                  7
                  31
                  97


                  =======================



                  The good primes up to 30,000 are



                  2
                  7
                  31
                  97
                  127
                  607
                  8191
                  12289
                  22783


                  ===========






                  share|cite|improve this answer









                  $endgroup$



                  This is the pigeonhole principle. Among odd primes $p,$ there are solutions to $2t^2 - 1 equiv 0 pmod p$ if and only if $p equiv pm 1 pmod 8.$



                  For such a prime $p,$ there are some $frac{p+1}{2}$ distinct values $pmod p$ of $2x^2 - 1.$ Therefore, taking $p geq 17,$ if we take, say, $p-5$ steps of your sequence, there are just two possibilities: either



                  (I) there is a repetition of some value $pmod p$ that is not one of $-1,0,1.$ In this case, the sequence repeats again and again, forever. Thus, this prime $p$ is never a factor of any of your $x_n.$ OR



                  (II) at least one of $-1,0,1$ occurs by the $p-5$ deadline. Note that $0$ must come first out of these three, as $2 cdot 1^2 - 1 equiv 2 cdot (-1)^2 - 1 equiv 1 pmod p,$ while the only way to get $-1$ is $2 cdot 0^2 - 1 equiv -1 pmod p.$



                  That's it. If $p$ will ever be a factor of any $x_n,$ it is because, say, $x_{p-2} equiv 1 pmod p$



                  jagy@phobeusjunior:~$ ./mse 
                  7
                  2 0 -1

                  17
                  2 7 12 15 7

                  23
                  2 7 5 3 17 2

                  31
                  2 7 4 0 -1

                  41
                  2 7 15 39 7

                  47
                  2 7 3 17 13 8 33 15 26 35
                  5 2

                  71
                  2 7 26 2

                  73
                  2 7 24 56 66 24

                  79
                  2 7 18 15 54 64 54

                  89
                  2 7 8 38 39 15 4 31 52 67
                  77 20 87 7

                  97
                  2 7 0 -1


                  good primes
                  2
                  7
                  31
                  97


                  =======================



                  The good primes up to 30,000 are



                  2
                  7
                  31
                  97
                  127
                  607
                  8191
                  12289
                  22783


                  ===========







                  share|cite|improve this answer












                  share|cite|improve this answer



                  share|cite|improve this answer










                  answered Dec 11 '18 at 2:25









                  Will JagyWill Jagy

                  104k5102201




                  104k5102201






























                      draft saved

                      draft discarded




















































                      Thanks for contributing an answer to Mathematics Stack Exchange!


                      • Please be sure to answer the question. Provide details and share your research!

                      But avoid



                      • Asking for help, clarification, or responding to other answers.

                      • Making statements based on opinion; back them up with references or personal experience.


                      Use MathJax to format equations. MathJax reference.


                      To learn more, see our tips on writing great answers.




                      draft saved


                      draft discarded














                      StackExchange.ready(
                      function () {
                      StackExchange.openid.initPostLogin('.new-post-login', 'https%3a%2f%2fmath.stackexchange.com%2fquestions%2f3030131%2fprove-that-for-all-n-gcdn-x-n-1-given-x-n1-2x-n2-1-and-x-1-2%23new-answer', 'question_page');
                      }
                      );

                      Post as a guest















                      Required, but never shown





















































                      Required, but never shown














                      Required, but never shown












                      Required, but never shown







                      Required, but never shown

































                      Required, but never shown














                      Required, but never shown












                      Required, but never shown







                      Required, but never shown







                      Popular posts from this blog

                      Biblatex bibliography style without URLs when DOI exists (in Overleaf with Zotero bibliography)

                      ComboBox Display Member on multiple fields

                      Is it possible to collect Nectar points via Trainline?